Board logo

标题: 求高手解决一道题目 [打印本页]

作者: changjiangtao    时间: 2011-4-27 07:06     标题: 求高手解决一道题目

48. A researcher discovered that people who have low levels of immune-system activity tend to score much lower on tests of mental health than do people with normal or high immune-system activity. The researcher concluded from this experiment that the immune system protects against mental illness as well as against physical disease



.
The researcher’s conclusion depends on which of the following assumptions?



A. High immune-system activity protects against mental illness better than normal immune-system activity does.



B. Mental illness is similar to physical disease in its effects on body systems.



C. People with high immune-system activity cannot develop mental illness.



D. Mental illness does not cause people’s immune-system activity to decrease.D



E. Psychological treatment of mental illness is not as effective as is medical treatment.

题目不是说immune-system activity 低的人在测试中的成绩低,研究员的结论就是the immune system protects 对于mental illness 和 physical disease 的作用是一样的。题目问研究员的结果是依据下面那个假设的?
那我就觉得结果应该是B呀,为什么答案是D,D是怎么退出来的

作者: newolder    时间: 2011-4-27 21:43

觉得B不对,body system 和immune system范围不同,D起码沾边
作者: changjiangtao    时间: 2011-4-28 06:31

但是D根本没有解释为什么physical disease的作用是一样的呀
作者: supermanlee    时间: 2011-4-29 21:37

Necessary assumption. Use negation.

If you negate D, mental illness does cause people’s immune-system activity to decrease, then the immune system cannot protect people's mental health. The conclusion of the stimulus falls apart.

P.S. It is common sense that immune system protects physical health.
作者: changjiangtao    时间: 2011-4-30 06:41

谢谢帮助
作者: natalietong    时间: 2011-5-2 09:22

去掉not之后是因果倒置的削弱




欢迎光临 国际顶尖MBA申请交流平台--TOPWAY MBA (http://forum.topway.org/) Powered by Discuz! 7.2